English Logic Puzzle












0












$begingroup$


On this site, I found this:




"If some Smaugs are Thors and some Thors are Thrains, then some Smaugs are definitely Thrains."




Options: TRUE, FALSE, NEITHER



The answer given is FALSE. However, I need convincing that the answer isn't 'NEITHER'. How can I see this?



(I got all the rest right btw)









share









$endgroup$

















    0












    $begingroup$


    On this site, I found this:




    "If some Smaugs are Thors and some Thors are Thrains, then some Smaugs are definitely Thrains."




    Options: TRUE, FALSE, NEITHER



    The answer given is FALSE. However, I need convincing that the answer isn't 'NEITHER'. How can I see this?



    (I got all the rest right btw)









    share









    $endgroup$















      0












      0








      0





      $begingroup$


      On this site, I found this:




      "If some Smaugs are Thors and some Thors are Thrains, then some Smaugs are definitely Thrains."




      Options: TRUE, FALSE, NEITHER



      The answer given is FALSE. However, I need convincing that the answer isn't 'NEITHER'. How can I see this?



      (I got all the rest right btw)









      share









      $endgroup$




      On this site, I found this:




      "If some Smaugs are Thors and some Thors are Thrains, then some Smaugs are definitely Thrains."




      Options: TRUE, FALSE, NEITHER



      The answer given is FALSE. However, I need convincing that the answer isn't 'NEITHER'. How can I see this?



      (I got all the rest right btw)







      english logic-theory





      share












      share










      share



      share










      asked 4 mins ago









      JonMark PerryJonMark Perry

      18.8k63890




      18.8k63890






















          0






          active

          oldest

          votes











          Your Answer





          StackExchange.ifUsing("editor", function () {
          return StackExchange.using("mathjaxEditing", function () {
          StackExchange.MarkdownEditor.creationCallbacks.add(function (editor, postfix) {
          StackExchange.mathjaxEditing.prepareWmdForMathJax(editor, postfix, [["$", "$"], ["\\(","\\)"]]);
          });
          });
          }, "mathjax-editing");

          StackExchange.ready(function() {
          var channelOptions = {
          tags: "".split(" "),
          id: "559"
          };
          initTagRenderer("".split(" "), "".split(" "), channelOptions);

          StackExchange.using("externalEditor", function() {
          // Have to fire editor after snippets, if snippets enabled
          if (StackExchange.settings.snippets.snippetsEnabled) {
          StackExchange.using("snippets", function() {
          createEditor();
          });
          }
          else {
          createEditor();
          }
          });

          function createEditor() {
          StackExchange.prepareEditor({
          heartbeatType: 'answer',
          autoActivateHeartbeat: false,
          convertImagesToLinks: false,
          noModals: true,
          showLowRepImageUploadWarning: true,
          reputationToPostImages: null,
          bindNavPrevention: true,
          postfix: "",
          imageUploader: {
          brandingHtml: "Powered by u003ca class="icon-imgur-white" href="https://imgur.com/"u003eu003c/au003e",
          contentPolicyHtml: "User contributions licensed under u003ca href="https://creativecommons.org/licenses/by-sa/3.0/"u003ecc by-sa 3.0 with attribution requiredu003c/au003e u003ca href="https://stackoverflow.com/legal/content-policy"u003e(content policy)u003c/au003e",
          allowUrls: true
          },
          noCode: true, onDemand: true,
          discardSelector: ".discard-answer"
          ,immediatelyShowMarkdownHelp:true
          });


          }
          });














          draft saved

          draft discarded


















          StackExchange.ready(
          function () {
          StackExchange.openid.initPostLogin('.new-post-login', 'https%3a%2f%2fpuzzling.stackexchange.com%2fquestions%2f79103%2fenglish-logic-puzzle%23new-answer', 'question_page');
          }
          );

          Post as a guest















          Required, but never shown

























          0






          active

          oldest

          votes








          0






          active

          oldest

          votes









          active

          oldest

          votes






          active

          oldest

          votes
















          draft saved

          draft discarded




















































          Thanks for contributing an answer to Puzzling Stack Exchange!


          • Please be sure to answer the question. Provide details and share your research!

          But avoid



          • Asking for help, clarification, or responding to other answers.

          • Making statements based on opinion; back them up with references or personal experience.


          Use MathJax to format equations. MathJax reference.


          To learn more, see our tips on writing great answers.




          draft saved


          draft discarded














          StackExchange.ready(
          function () {
          StackExchange.openid.initPostLogin('.new-post-login', 'https%3a%2f%2fpuzzling.stackexchange.com%2fquestions%2f79103%2fenglish-logic-puzzle%23new-answer', 'question_page');
          }
          );

          Post as a guest















          Required, but never shown





















































          Required, but never shown














          Required, but never shown












          Required, but never shown







          Required, but never shown

































          Required, but never shown














          Required, but never shown












          Required, but never shown







          Required, but never shown







          Popular posts from this blog

          What are all the squawk codes?

          What are differences between VBoxVGA, VMSVGA and VBoxSVGA in VirtualBox?

          Hudsonelva